PETH 2

अब Quizwiz के साथ अपने होमवर्क और परीक्षाओं को एस करें!

Which of the following situations, during the period of the professional engagement, would definitely impair a member's independence with an attest client? A) The member had an immaterial indirect interest in the client. B) The member had an immaterial direct interest in the client. C) The member participated on the client's advisory board that acted in a purely advisory capacity. D) All of the above would definitely impair independence.

Answer: B) any direct interest regardless of immateriality impairs independence Immaterial indirect financial interests will not impair independence. Conversely, material indirect financial interests will impair independence. Since there is no materiality with respect to direct financial interests, the following investments of as little as one share of stock will impair independence:

Romine is a CPA and an audit manager at Rivera & Rivera, CPAs. Romine has worked for Rivera & Rivera, CPAs for ten years. Because Romine is considered to be an expert in audits of public universities, he has been asked by Local State University to be an adjunct faculty member and to design and teach a new class in audits of public universities. Local State University is a client of Rivera & Rivera, CPAs and has been for the past five years.Which of the following would not impair Rivera & Rivera, CPAs independence with respect to Local State University? A) Romine participates on the attest engagement team for the Local State University audit B) Romine participates on a committee that sets policies for Local State University C) Romine is employed by the university on a part-time and nontenure basis D) None of the above would impair independence

Answer: C .03 However, threats will be at an acceptable level and independence will not be impaired if a partner or professional employee of a firm serves as an adjunct faculty member of an educational institution that is an attest client of the firm, provided that the partner or professional employee meets all of the following safeguards: a. Does not hold a key position at the educational institution b. Does not participate on the attest engagement team c. Is not an individual in a position to influence the attest engagement d. Is employed by the educational institution on a part-time and non-tenure basis** e. Does not participate in any employee benefit plans sponsored by the educational institution, unless participation is required f. Does not assume any management responsibilities or set policies for the educational institution

Sherman & Hirsch, CPAs were engaged on December 1, 20X2 to audit the financial statements of Stearn Financial, Inc. for the year ended December 31, 20X2. While doing an internal investigation to determine whether any issues and/or threats to independence exist within the firm as it relates to the audit of Stearn Financial, Inc., the firm was able to satisfy itself on all but one issue. Upon reviewing the Code of Professional Conduct, and specifically Rule 101 regarding independence, the firm was unable to find any interpretations or rulings regarding the one issue in question. Which of the following statements is true regarding how Sherman & Hirsch, CPAs should handle the one issue of independence that is in question? A) When there is no interpretation or ruling that covers the one issue of independence in question, the firm may assume that the issue was not important enough to be covered and thus does not impair independence. B) When there is no interpretation or ruling that covers the one issue of independence in question, the firm should send a written request to the AICPA to receive a private ruling on the issue involved. The firm should not proceed with the audit until it receives the private ruling. C) When there is no interpretation or ruling that covers the one issue of independence in question, the firm should use the conservative approach and assume that the issue in question would impair the firm's independence. D) The firm should evaluate whether that circumstance would lead a reasonable person aware of all the relevant facts to conclude that there is an unacceptable threat to the firm's independence.

Answer: D) It is not possible to have the rules, interpretations and rulings cover all the possible situations that might impair a member's independence. In those situations where the matter is not specifically addressed by the rules, interpretations or rulings, members should use this conceptual framework to decide whether independence or the appearance of independence is impaired.

Which of the following constitutes the receipt of a commission? A) A member contracts with a service provider to provide services to the member's client and the member charges the client more than the service provider charges the member. B) Purchasing a product from a third-party supplier and reselling it to a client at a profit. C) Both a. and b. D) Neither a. or b.

Answer: D) neither a nor b 1.520.060 Sale of Products to Clients .01 If a member purchases a product, taking title to the product and assuming all the associated risks of ownership, any profit the member receives on reselling it to a client would not constitute a commission. 1.520.070 Billing for a Subcontractor's Services .01 If, in providing professional services to a client,a member subcontracts the services of another person or entity, any mark-up of the cost of the subcontracted services would not constitute a commission.

Integrity requires a member to observe which of the following principles? A) Objectivity and independence B) Due care C) Generally Accepted Accounting Principles D) All of the above E) Both a. and b.

Answer: E) both a and b Integrity also requires a member to observe the principles of objectivity and independence and of due care.

Su & Wong, CPAs is not independent with respect to Radar Corporation because the firm provides bookkeeping services for the client and makes management decisions regarding check coding. Zee & Brock, CPAs is performing an attest engagement for Radar Corporation and is using professional staff from Su & Wong, CPAs on the engagement.Is Zee & Brock, CPAs' independence with respect to Radar Corporation impaired due to the use of Su & Wong, CPAs' employees on the attest engagement? A) Yes, independence is impaired. B) No, independence is not impaired. C) Independence is only impaired if a partner from Su & Wong, CPAs participates on the attest engagement. D) Independence is only impaired if Su & Wong, CPAs are included within the attest engagement letter with Radar Corporation

A) Yes, independence is impaired Yes. The Independence of Zee &. Brock is impaired. If a member uses the services of another partner with non attest relationship with the client that has significantly impaired the Independence of that firm, their Independence is also impaired. Step-by-step explanation The principle of Independence prevents a member from offering services that ultimately involves the same financials that they're auditing. Partnering with a member whose Independence is impaired also affects the firm offering attest services. Source https://pcaobus.org/Standards/EI/Pages/ET101.aspx 05 Attest engagement team. Those individuals participating in the attest engagement, including those who perform concurring and engagement quality reviews. The attest engagement team includes all employees and contractors retained by the firm who participate in the attest engagement, regardless of their functional classification (for example, audit, tax, or management consulting services). The attest engagement team excludes specialists, as discussed in AU-C section 620, Using the Work of an Auditor's Specialist (AICPA, Professional Standards), and individuals who perform only routine clerical functions, such as word processing and photocopying. [Prior reference: paragraph .02 of ET section 92]

Which of the following is not a characteristic of network firms? - independent business strategies amongst the firms in the network - sharing significant business resources - sharing profits or costs - a common brand name

A) not independent strategies - sharing strategies Characteristics of a Network .06 When an association is formed for the purpose of cooperating to enhance the firms' capabilities to provide professional services, and one of the characteristics described in paragraphs .07-.18 of this section also applies, the association is considered to be a network. .07 Sharing a common brand name. This characteristic exists when the association's members or entities controlled by the association's members share the use of a common brand name or share common initials as part of the firm name. .08 A firm that does not use a common brand name as part of its firm name but makes reference in its stationery or promotional materials to being a member of an association of firms should carefully consider how it describes that membership and take steps to avoid the perception that it belongs to a network. The firm may wish to avoid such perception by clearly describing the nature of its membership in the association (for example, by stating on its stationery or promotional material that it is "an independently owned and operated member firm of XYZ Association"). .09 Sharing common control. This characteristic exists when entities within the association are under common control with other firms in the association through ownership, management, or other means (for example, by contract). However, compliance with association requirements as a condition of membership does not indicate that members are under common control; rather, it reflects the type of cooperation that is expected when an entity joins the association. .10 Sharing profits or costs. This characteristic exists when entities within the association share profits or costs. Following are examples of profit and cost sharing that would not create a network: Sharing immaterial costs Sharing costs related to operating the association Sharing costs related to the development of audit methodologies, manuals, and training courses Arrangements between a firm and an otherwise unrelated entity to jointly provide a service or develop a product .11 Sharingacommonbusinessstrategy.Thischaracteristic exists when entities within the association share a common business strategy. Sharing a common business strategy involves ongoing collaboration among the firms whereby the firms are responsible for implementing the association's strategy and held accountable for performance pursuant to that strategy. An entity's ability to pursue an alternative strategy may be limited by the common business strategy because, as a member, it must act in accordance with the common business strategy and, therefore, in the best interest of the association. .12 An entity is not considered to be a network firm merely because it cooperates with another entity solely to market professional services or responds jointly toa request for a proposal for the provision of a professional service. .13 Sharing significant professional resources.This characteristic exists when entities within the association share a significant part of professional resources. Members should consider both qualitative and quantitative factors in determining whether the shared professional resources are significant.

Marlon, a member, received his CPA license three years ago and has worked for a local CPA firm for the past 5 years. Marlon recently decided to start his own CPA practice and wants to employ various marketing and promotional techniques to facilitate growth. Which of the following would be a violation of the Code's rules regarding advertising and solicitation? A) The use of telemarketers who have been told to say that Marlon has been a CPA for 5 years and has worked in public accounting for 10 years. B) The use of a local newspaper ad that states that Marlon is opening his CPA practice and is seeking new business. C) Both a. and b. D) Neither a. nor b.

A). false advertising .01 A member in public practice shall not seek to obtain clients by advertising or other forms of solicitation in a manner that is false, misleading, or deceptive. Solicitation by the use of coercion, over-reaching, or harassing conduct is prohibited. [Prior reference: paragraph .01 of ET section 502]

Which of the following is an example of an "other member" according to the Code? - an unemployed member - a non-member - a member who is both in business and in public practice - a nonmember CPA candidate - none of the above

A. an unemployed member Who of the following is an "other member" for code purposes? all of the above,Tim, who is a retired CPATom, who is an unemployed CPABill, who is a CPA currently working as a rodeo clown Members often serve multiple interests in many different capacities and must demonstrate their objectivity in varying circumstances. Members in public practice render attest, tax, and management advisory services. .vs. Other members prepare financial statements in the employment of others, perform internal auditing services, and serve in financial and management capacities in industry, education, and government. They also educate and train those who aspire to admission into the profession. Regardless of service or capacity, members should protect the integrity of their work, maintain objectivity, and avoid any subordination of their judgment. https://www.aicpa.org/content/dam/aicpa/research/standards/codeofconduct/downloadabledocuments/2014december14codeofprofessionalconduct.pdf

Harkin & Larkin, CPAs is a multi-office firm with offices located in Phoenix and Scottsdale, Arizona. Moss, who is a manager in the Phoenix office, serves on the Board of Directors of Marsden Corporation, a company that is not currently a client of the firm.Marsden Corporation has asked Moss if his firm would be interested in performing the audit of the company's financial statements for the year ended December 31, 20X6. Moss knows that if the Phoenix office handles the audit or if Moss is a member of the audit team, the firm's independence would be impaired. However, Moss believes that if the Scottsdale office handles the audit, and if he does not participate in the audit nor provide any other services to the company, the firm's independence would not be impaired.Moss approaches Joe Harkin, the managing partner of Harkin & Larkin, CPAs, to seek his advice regarding the situation. Which of the following responses from Joe Harkin would be appropriate? A) Moss is correct that if the Scottsdale office handles the audit, and if Moss does not participate in any way on the audit team or provide any services to Marsden Corporation, the firm's independence is not impaired. B) It is ok for the Phoenix office to handle the audit as long as Moss does not participate in any way with the audit team or provide any services to Marsden Corporation. The firm's independence would not be impaired. C) While Moss is not a covered member if he does not work in the office handling the audit and if he performs no services for Marsden Corporation, the firm's independence is impaired because he serves in a capacity equivalent to a member of management. D) None of the above

ANS: B okay because moss is only a MANAGER not a partner in a different office not involded with the audit Who is a covered member? .12 Covered member. All of the following: a. an individual on the attest engagement team. b. an individual in a position to influence the attest engagement. c. a partner, partner equivalent, or manager who provides more than 10 hours of nonattest services to the attest client within any fiscal year.Designation as covered member ends on the laterof (i) the date that the firm signs the report on the financial statements for the fiscal year during which those services were provided or (ii) the date he or she no longer expects to provide 10 or more hours of nonattest services to the attest client on a recurring basis. d. a partner or partner equivalent in the office in which the lead attest engagement partner or partner .08 .09 equivalent primarily practices in connection with the attest engagement. the firm, including the firm's employee benefit plans. an entity whose operating, financial, or accounting policies can be controlled by any of the individuals or entities described in items a-e or two or more such individuals or entities if they act together. [Prior reference: paragraph .07 of ET section 92] Effective Date The addition of partner equivalents to this definition is effective for engagements covering periods beginning on or after December 15, 2014.

Salmon & Trout, CPAs perform audits for several publicly traded companies that file their financial statements with the Securities and Exchange Commission.Under the Securities and Exchange Commission rules, which of the following would impair Salmon & Trout, CPA's independence with respect to the noted audit client? A) A professional employee of Salmon & Trout, CPA's is simultaneously employed by one of the firm's SEC audit clients. B) A partner in the firm of Salmon & Trout, CPAs has a savings account with an SEC audit client. The balance in the savings account is below the amount insured by the Federal Deposit Insurance Corporation. C) A manager in the firm of Salmon & Trout, CPAs has an automobile loan with one of the firm's SEC audit clients. The loan was obtained under the audit client's normal lending procedures, terms, and requirements. The loan is fully collateralized by the automobile. D) The firm of Salmon & Trout, CPAs, upon request by the audit client, interviews candidates and advises the audit client on the candidate's competence for financial accounting, administrative, or control positions. The firm does not recommend that the audit client hire a specific candidate for a specific job.

Answer: A) (2) Employment relationships. An accountant is not independent if, at any point during the audit and professional engagement period, the accountant has an employment relationship with an audit client, such as: (i) Employment at audit client of accountant. A current partner, principal, shareholder, or professional employee of the accounting firm is employed by the audit client or serves as a member of the board of directors or similar management or governing body of the audit client. (B) Savings and checking accounts. Any savings, checking, or similar account at a bank, savings and loan, or similar institution that is an audit client, if the account has a balance that exceeds the amount insured by the Federal Deposit Insurance Corporation or any similar insurer, except that an accounting firm account may have an uninsured balance provided that the likelihood of the bank, savings and loan, or similar institution experiencing financial difficulties is remote. Loan/debtor-creditor relationship. Any loan (including any margin loan) to or from an audit client, or an audit client's officers, directors, or record or beneficial owners of more than ten percent of the audit client's equity securities, except for the following loans obtained from a financial institution under its normal lending procedures, terms, and requirements: (1) Automobile loans and leases collateralized by the automobile; (E) Recommending, or advising the audit client to hire, a specific candidate for a specific job (except that an accounting firm may, upon request by the audit client, interview candidates and advise the audit client on the candidate's competence for financial accounting, administrative, or control positions).

Sandoval Research is required to file its audited financial statements with the Government Accountability Office. Cain & Scutaro, CPAs has been hired to perform the engagement for the year ending December 31, 20X7. Cain & Scutaro, CPAs has identified several threats to its independence with respect to Sandoval Research. Which of the following potential safeguards might be effective in reducing the threat or threats to an acceptable level? A) Consulting with an independent third party B) Having a professional staff member who was a member of the audit team review the work performed C) Allowing a member of the audit team to have a financial interest in the audit client as long as the interest is less than 10% of the staff member's net worth. D) All of the above

Answer: A) 3.17 Examples of safeguards include: a. consulting an independent third party, such as a professional organization, a professional regulatory body, or another auditor;** b. involving another audit organization to perform or re-perform part of the audit; c. having a professional staff member who was NOT a member of the audit team review the work performed; and d. removing an individual from an audit team when that individual's financial or other interests or relationships pose a threat to independence.

Gattis, a member, performed an attest engagement for Brave Industries, Inc. for the year ended December 31, 20X6. The report on the attest engagement was issued on February 7, 20X7. A regulatory body that Brave Industries, Inc. is governed by sent a letter to Gattis on June 23, 20X7 requesting confidential information on the company in connection with their investigation of Brave Industries, Inc. Which of the following statements is true regarding Gattis' disclosure of the requested confidential information? A) Gattis is required to comply with the request of any regulatory body. B) If Brave Industries, Inc. refuses to give Gattis permission to disclose the confidential information, Gattis may not disclose the confidential client information. C) Gattis does not have to comply with the request of the regulatory body under any circumstances since the regulatory body does not govern Gattis' actions. D) None of the above

Answer: A) Section 54.1. Disclosure of Confidential Information Prohibited (a) No confidential information obtained by a licensee, in his or her professional capacity, concerning a client or a prospective client shall be disclosed by the licensee without the permission of the client or prospective client, except for the following: (1) disclosures made by a licensee in compliance with a subpoena or a summons enforceable by order of a court; (2) disclosures made by a licensee regarding a client or prospective client to the extent that the licensee reasonably believes that it is necessary to maintain or defend himself/herself in a legal proceeding initiated by that client or prospective client; (3) disclosures made by a licensee in response to an official inquiry from a federal or state government regulatory agency;**** (4) disclosures made by a licensee or a licensee's duly authorized representative to another licensee in connection with a proposed sale or merger of the licensee's professional practice; (5) disclosures made by a licensee to (A) another licensee to the extent necessary for purposes of professional consultation and to (B) professional standards review, ethics or quality control peer review organizations; (6) disclosures made when specifically required by law; (7) disclosures made at the direct request of the client to a person or entity that is designated by the client at the time of the request.

Rosson, a California CPA, was a partner with the California firm of Freedman & Warwick, CPA's. She worked for the firm for forty-eight years, thirty-five of them as a partner and the last six as managing partner. When Rosson retired from the firm, the other partners voted to pay her a retirement annuity of $100,000, which exceeded the $75,000 she was entitled to per the partnership agreement. It was the other partners' belief that Rosson had provided exemplary service and deserved the additional amount in her retirement annuity. Is this arrangement allowed by the California Accountancy Act? A) Yes, it is allowed by the California Accountancy Act. B) No, this arrangement would be a commission and would not be permitted. C) Yes, it is allowed as long as the annual payment does not exceed $100,000. D) No, it is not allowed because the annual payment exceeds $50,000.

Answer: A) Yes it is allowed by the Act When Leung retired as managing partner of "Leung and Chang, CPAs," she was entitled to a retirement annuity of $75,000 per year until she died. Is this arrangement allowed by the California Accountancy Act? Yes, it is allowed by the California Accountancy Act. No, this arrangement would be a commission and would not be permitted. Yes, it is allowed as long as the annual payment does not exceed $100,000. No, it is not allowed because the annual payment exceeds $50,000. Correct answer is a.

McDaddy, a member, prepared a personal tax return for Ferguson for the year 20X4 which was filed electronically on April 15, 20X5. In September 20X5, the 20X4 return was audited and the examination resulted in the disallowance of Deduction A. The examination concluded on November 1, 20X5.In March of 20X6, McDaddy is in the process of preparing a personal tax return for Ferguson for the year 20X5 and Ferguson wants McDaddy to include Deduction A in the tax return. Which of the following statements is true regarding McDaddy's responsibilities as it relates to taking the previously disallowed Deduction A on Ferguson's 20X5 tax return? A) McDaddy is bound by the disallowance of Deduction A on the 20X4 tax return and is not allowed to sign a return for 20X5 that contains Deduction A. B) If the disallowance of Deduction A was caused by a lack of documentation, McDaddy may recommend taking Deduction A and can sign the return. C) Since the taxing authority will always rule in the same manner regarding Deduction A as it relates to Ferguson, McDaddy is bound by the disallowance of Deduction A on the 20X4 tax return and is not allowed to sign a return for 20X5 that contains Deduction A. D) None of the above

Answer: B .02 A member should determine and comply with the reporting and disclosure standards, if any, that are imposed by the applicable taxing authority with respect to recommending a tax return position or preparing or signing a tax return. If the applicable taxing authority has no written standards that apply with respect to recommending a tax return position or preparing or signing a tax return or if its standards are lower than the standards set forth in this paragraph, the following standards will apply: a. A member should not recommend a tax return position or prepare or sign a tax return taking a position unless the member has a good-faith belief that the position has at least a realistic possibility of being sustained administratively or judicially on its merits, if challenged (commonly referred to as the realistic possibility of success standard). b. Notwithstanding paragraph 2(a), a member may recommend a tax return position if the member (i) concludes that there is a reasonable basis for the position, and (ii) advises the taxpayer to appropriately disclose that position. Notwithstanding paragraph 2(a), a member may prepare or sign a tax return that reflects a position if (i) the member concludes there is a reasonable basis for the position, and (ii) the position is appropriately disclosed. - checked

Daniel, a member, is a CPA and has been licensed since 20X1. In 20X4, Daniel decided to attend law school and was admitted to his State Bar in 20X7. Daniel maintains both licenses and currently practices as both a CPA and a lawyer. According to the Code of Professional Conduct, which of the following is true? A) The Code explicitly prohibits the simultaneous practice of accounting and law by a member licensed in both professions. B) The Code does not explicitly prohibit the simultaneous practice of accounting and law by a member licensed in both professions. C) The Code allows Daniel to represent himself on his letterhead as both a CPA and an attorney. D)The Code explicitly states that a member should refer to applicable state accountancy laws and board of accountancy rules and regulations for guidance regarding the use of the CPA credential and, as such, the issue is not explicitly addressed in the Code

Answer: B) 78. Letterhead: Lawyer-CPA .155 Question—May a member who is also admitted to the Bar represent him or herself on his or her letterhead as both an attorney and a CPA, or should he or she use separate letterheads in the conduct of the two practices? .156 Answer—The Code does not prohibit the simultaneous practice of accounting and law by a member licensed in both professions. Either a single or separate letterheads may be used, provided the information with respect to the CPA designation complies with rule 502 [ET section 502.01]. However, the member should also consult the rules of the applicable Bar Association. https://www.aicpa.org/research/standards/codeofconduct/downloadabledocuments/2011june1codeofprofessionalconduct.pdf

Miley, a covered member, prepared the financial statements for Cyrus Productions for the year ended December 31, 20X5. On May 1, 20X7, First Bank filed a third-party lawsuit against Miley alleging reliance on the financial statements of Cyrus Productions with which Miley is associated as a basis for extending credit to the company.Which of the following statements is true regarding Miley's independence with regard to Cyrus Productions, Inc.? A) Independence is always impaired between the covered member (defendant) and the client in these situations. B) If the client is not a party to the lawsuit, independence is not normally impaired. C) Independence is never impaired between the covered member (defendant) and the client in these situations. D) If the covered member, in their defense, alleges fraud or deceit by the present management of the company, independence is not impaired.

Answer: B) D is true if it said independence IS impaired Other Third-Party Litigation .12 A lending institution or other creditor, security holder, or insurance company that alleges reliance on the attest client's financial statements as a basis for having extended credit or insurance coverage to an attest client may commence third-party litigation against the covered member to recover their loss. An example is an insurance company commencing litigation either as a result of receiving an assignment of a claim or under subrogation rights against the covered member in the attest client's name to recover losses that the insurer reimbursed to the attest client. If the attest clientis only the nominal plaintiff, threats to the covered member's compliance with the "Independence Rule" [1.200.001] would be at an acceptable level unless other circumstances exist, such as when the covered member alleges, as a defense, that present management engaged in fraud or deceit. The attest client is a nominal plaintiff when the insurance company or lender suesin the name of the attest client as a result of obtaining subrogation rights or an assignment from the attest client and the attest client does not have a beneficial interest in the claim.

Susan Cue is a CPA who is not in public practice. Susan is employed as the Chief Financial Officer for Fogerty Enterprises and has been for the past ten years. Prior to working for Fogerty Enterprises, Susan worked in public accounting for the firm of Creedence & Clearwater, CPAs. When Susan left Creedence & Clearwater, CPAs, she was an audit manager. Fogerty Enterprises is a client of Creedence & Clearwater, CPAs during Susan's employment with the firm and continues to be an audit client of the firm.Susan Cue wants to use her CPA designation on her business cards with Fogerty Enterprises. Which of the following may Susan Cue use on her business cards? A) Susan Cue, CPA, formerly with Creedence & Clearwater, CPAs B) Susan Cue, CPA, Chief Financial Officer, Fogerty Enterprises C) Susan Cue, CPA D) Susan Cue, CPA, (not in public practice)

Answer: B) Title of Certified Public Accountant Section 5055. Any person who has received from the board a certificate of certified public accountant, or who is authorized to practice public accountancy in this state pursuant to Article 5.1 (commencing with Section 5096) may, subject to Section 5051, be styled and known as a "certified public accountant" and may also use the abbreviation "C.P.A." No other person, except a firm registered under this chapter, shall assume or use that title, designation, or abbreviation or any other title, designation, sign, card or device tending to indicate that the person using it is a certified public accountant.

Dana Solomon is a partner in the firm Seidner & Wilner, CPAs. The firm is located in Santa Monica, California and has a total of 25 professional staff. The firm prepared the audited financial statements of Hamilton, Inc. for the year ended December 31, 20X1 and issued its report on February 18, 20X2. Hamilton, Inc. is a publicly traded corporation. Dana Solomon was the lead partner in charge of the Hamilton, Inc. audit for the year ended December 31, 20X1. Dana Solomon has been offered a position as the Chief Financial Officer of Hamilton, Inc. Under the California Accountancy Act, what is the earliest date that Dana Solomon may accept this position? A) January 1, 20X2 B) February 19, 20X3 C) January 1, 20X3 D) February 19, 20X2

Answer: B) at least a year from issue of the report = feb 19 20x3 The law provides for partner rotation on the audit team. It also provides a cooling off period of a year before audit engagement partners may be hired by the client in a financial oversight role.

Alex, a member, provides various attest and non-attest services to Bisk & Associates and has been doing so for approximately fifteen years. During this fifteen year association with Bisk & Associates there have been no major disagreements or disputes between the parties and Alex has always provided timely and responsive services to the client.Alex prepared a financial statement for Bisk & Associates for the year 20X2 which was submitted to Bisk & Associates' bank for the purpose of obtaining credit. The financial statement was completed by Alex in February 20X3 and submitted to the bank in March 20X3.In May 20X3, Bisk & Associates discovered that their bookkeeper had committed fraud and had stolen approximately $100,000 from the company over a three year period. As a result of this discovery, Bisk & Associates initiated a claim for malpractice against Alex. When Alex was informed of the claim she immediately contacted her professional liability insurance carrier as required by the carrier.Which of the following is a violation of the prohibition against disclosure of confidential client information? A) Complying with the insurance carrier's request for Bisk & Associates' general ledger B) Reviewing all correspondence and emails between Alex and Bisk & Associates for the past three years with a professional colleague who had been previously sued for malpractice by one of their clients.C) Complying with the insurance carrier's request for all correspondence and emails between Alex and Bisk & Associates for the past three years D) Both a. and c. are violations of the prohibition against disclosure of confidential client information. E) Both b. and c. are violations of the prohibition against disclosure of confidential client information.

Answer: B) is a violation Members' professional liability insurance policies often contain a requirement that the carrier be promptly notified of any claim or potential claim against the member.Upon receiving such notice the insurance company may request documents relating to the matter. These documents may contain confidential client information. Compliance by the member in furnishing such material to the carrier,s olely to assist in the defense of an actual or potential claim, is not a violation of the Code of Professional Conduct.

Lynn Hanna, a member, is a manager on the accounting staff of Duffy Construction, Inc. Hay & Ho, CPAs is performing an audit of the Duffy Construction, Inc. financial statements for the year ended June 30, 20X2 and is in the final stages of the audit. As part of their audit procedures, Mike Simmonds, the audit manager in charge of the Duffy Construction, Inc. audit, is gathering various representations, both written and oral, from various senior accounting personnel at Duffy Construction, Inc. Lynn Hanna works under the supervision of Dana Johnson, the controller of Duffy Construction, Inc. Jordan Walden is the CFO of Duffy Construction, Inc. and oversees all aspects of the accounting function for the company.Lynn Hanna is aware of a number of accounting irregularities that could result in a material misstatement of the Duffy Construction, Inc. financial statements. However, the actions that led to these irregularities, while actually performed by Lynn, were performed because Dana Johnson, the controller of the company, asked Lynn to do it based on orders received from Jordan Walden, the CFO of the company. Lynn was told by Dana Johnson that if she disclosed these irregularities to anybody she would be immediately terminated from her position with the company.Mike Simmonds has asked Lynn Hanna if she is aware of any items that have not been disclosed to the auditors that would have a material effect on the financial statements of Duffy Construction, Inc. With regard to this inquiry from Mike Simmonds, which of the following statements is correct? A) Lynn Hanna cannot disclose the information regarding the accounting irregularities because it would violate the rules regarding confidential company information. B) Lynn Hanna should not disclose the information regarding the accounting irregularities because she was told that such disclosure would result in her termination from her position with the company. C) Lynn Hanna is required to disclose the information regarding the accounting irregularities to Mike Simmonds. D) Lynn Hanna is not required to disclose the information regarding the accounting irregularities because she should assume that they will be disclosed by either Dana Johnson, the controller, or Jordan Walden, the CFO.

Answer: C Association With Misleading Information .09 If the member knows or has reason to believe thatthe information with which he or she is associatedis misleading, the member should apply appropriate safeguards to seek to resolve the matter, including the following: a. Consulting the employing organization's policies and procedures (for example, an ethics or whistleblowing policy) regarding how such matters should be addressed internally b. Discussing concerns that the information is misleading with the member's supervisor or the appropriate levels of management within the member's employing organization or those charged with governance and requesting such individuals to take appropriate action to resolve the matter. Such action may include the following: i. Having the information corrected ii. If the information has already been disclosed to the intended users, informing them of the correct information .10 If the member determines that appropriate actionhas not been taken and continues to have reason to believe that the information is misleading, threats to compliance with the "Integrity and Objectivity Rule" [2.100.001] would not be at an acceptable level.In such circumstances, the member, being alert to the requirements of the "Confidential Information Obtained From Employment or Volunteer Activities" interpretation [2.400.070], should consider one or more of the following safeguards: Part 2 — Members in Business 144 a. Consulting with a relevant professional body b. Consulting with the employing organization's internal auditor and external accountant c. Determining whether any requirements exist to communicate to third parties, including users of the information, the organization's external accountant, or regulatory authorities d. Consulting legal counsel regarding his or her responsibilities

Rolf, a member, is the CFO at Monopoly Inc., a public company. While in the restroom, she heard the CEO and COO having a heated conversation regarding the importance of "hitting their numbers" for the quarter and that it appeared as though, without manipulation, they will miss their targets. The CEO then tells the COO to "get people on board to get the numbers where we need them to be regardless of whether they are right or wrong." Rolf suspects that a fraud may have occurred or might occur. Which of the following actions is appropriate in this situation: - immediately report the situation to the SEC - immediately report the situation to the press - before pursuing a course of action, the member should consider consulting with appropriate persons within the organization that employs the member - none of the above

Answer: C) .05 In determining whether the pressure could result in a breach of the "Integrity and Objectivity Rule" [2.100.001], the member might consider factors including the following: a. The intent of the individual who is exerting the pressure and the nature and significance of the pressure. b. The application of relevant laws, regulations, and professional standards to the circumstances. c. The culture and leadership of the employing organization including the extent to which it emphasizes the importance of ethical behaviorand the expectation that employees will act in an ethical manner. For example, a corporate culture that tolerates unethical behavior may increase the likelihood that the pressure would result in a breach of the rules. d. Policies and procedures, if any, that the employing organization has established, such as ethics or human resources policies that address pressure. Ifthememberdeterminesthatthepressurewouldresult in a breach of the "Integrity and Objectivity Rule" [2.100.001], the member might consider safeguards, including these: a. Discussing the matter with the individual who is exerting the pressure to seek to resolve it. b. Discussing the matter with the member's supervisor if the supervisor is not the individual exerting the pressure. Part 2 — Members in Business 148 c. Escalating the matter within the employing organization, for example, with higher levels of management, internal or external auditors, or those charged with governance, including independent directors and, when appropriate, explaining any consequential risks to the organization.

The Conceptual Framework for AICPA Independence Standards describes the risk-based approach to analyzing independence matters. This approach requires which of the following steps? A) Utilizing a formal mathematical model to calculate risk and totally eliminating any threat to independence that exceeds a predetermined risk threshold. B) Utilizing a formal mathematical model to calculate risk and mitigating, to an acceptable level, any threat to independence that exceeds a predetermined risk threshold. C) Identifying and evaluating threats to independence. D) None of the above

Answer: C) In 2005 a Conceptual Framework for AICPA Independence Standards was issued by the Professional Ethics Executive Committee. In drafting Interpretation 101- 1, additional interpretations, and ethics rulings which specify certain relationships with clients that will impair independence and others which will not impair independence, the Professional Ethics Executive Committee uses the risk-based approach described in the Conceptual Framework. Using that, a member's relationship with a client is evaluated to determine whether it poses an unacceptable risk to the member's independence. It is unacceptable if it compromises or would be perceived to compromise the member's professional judgment.

Kershaw & Kemp, CPAs has offices in Los Angeles, CA and San Francisco, CA. The Los Angeles office handles the audit of Rodger Dodger Beverage Company. The San Francisco office does not perform any work and does not participate in the Rodger Dodger Beverage Company audit. Mattingly is a partner in the Los Angeles office and is the lead partner on the Rodger Dodger Beverage Company audit. Ethier is a partner in the Los Angeles office but is not involved in any way with the Rodger Dodger Beverage Company audit. Greinke works in the Los Angeles office and is the audit manager on the Rodger Dodger Beverage Company audit. Posey is a partner in the San Francisco office and is not in a position to influence the audit of Rodger Dodger Beverage Company.Which of the following relationships would, in itself, impair Kershaw & Kemp, CPA's independence with respect to the Rodger Dodger Beverage Company audit? A) Greinke's sister works as an administrative assistant for Rodger Dodger Beverage Company B) Ethier's daughter, who is not a dependent of Ethier, is the Chief Financial Officer of Rodger Dodger Beverage Company C) Mattingly's cousin is the controller of Rodger Dodger Beverage Company D) Posey's brother is a delivery person for Rodger Dodger Beverage Company

Answer: C) Mattingly = lead audit partner and covered member, plus family member in a key financial position .12 Covered member. All of the following: a. an individual on the attest engagement team. b. an individual in a position to influence the attest engagement. c. a partner, partner equivalent, or manager who provides more than 10 hours of nonattest services to the attest client within any fiscal year.Designation as covered member ends on the laterof (i) the date that the firm signs the report on the financial statements for the fiscal year during which those services were provided or (ii) the date he or she no longer expects to provide 10 or more hours of nonattest services to the attest client on a recurring basis. d. a partner or partner equivalent in the office in which the lead attest engagement partner or partner equivalent primarily practices in connection with the attest engagement. e. the firm, including the firm's employee benefit plans. f. an entity whose operating, financial, or accounting policies can be controlled by any of the individuals or entities described in items a-e or two or more such individuals or entities if they act together. [Prior reference: paragraph .07 of ET section 92] 9 Immediate family. A spouse, spousal equivalent,or dependent (regardless of whether the dependent is related). [Prior reference: paragraph .13 of ET section 92] 1.270.020 Immediate Family Member Is Employed by the Attest Client .01 When an individual in a covered member's immediate family is employed by an attest client, management participation, familiarity, and self-interest threats to the covered member's compliance with the "Independence Rule" [1.200.001] may exist. .02 If a covered member's immediate family is employed by an attest client but is not in a key position,threats would be at an acceptable level and independence would not be impaired. .03 If a covered member's immediate family is in a key position with an attest client during the period covered by the financial statements or during the period ofthe professional engagement, threats to compliance with the "Independence Rule" [1.200.001] would not be at an acceptable level and could not be reduced to an acceptable level by the application of safeguards. Accordingly, independence would be impaired.

Ron Ward, a member, is the sole owner of Ron Ward, CPA. Ron Ward has one bookkeeper and one accounting assistant working with him and neither is a CPA. Ron Ward, CPA prepares compiled financial statements for Packem Manufacturing Company and has done so for the past four years. Joe Packem, the owner of Packem Manufacturing Company, has asked Ron Ward if he has a client or is aware of anybody who would like to purchase some unneeded equipment from the company and has engaged the firm to help him find a buyer for this equipment. As part of the agreement, Packem Manufacturing Company will pay Ron Ward, CPA a commission of 5% of the selling price of the equipment if the firm finds a buyer and the sale is completed. At the present time, no buyer has been found and no sale has been completed.Which of the following statements is true regarding the performance of both the compilation and equipment selling engagements at the same time? A) Since the receipt of the commission is contingent upon the completion of a sale that has not yet been completed, there is no conflict between the two engagements and no further action must be taken by Ron Ward, CPA. B) Ron Ward, CPA cannot perform both of these services at the same time under any circumstances. C) If Ron Ward, CPA expects or reasonably expects that a third party will use the compiled financial statements, the report must disclose the firm's lack of independence. D) None of the above statements are true.

Answer: C) must disclose lack of independence for Compilations .01 Prohibited commissions. A member in public practice shall not for a commission recommend or refer to a client any product or service, or for a commission recommend or refer any product or service to be supplied by a client, or receive a commission, when the member or member's firm also performs for that client: a) an audit or review of a financial statement; or b) a compilation of a financial statement when the member expects, or reasonably might expect, that a third party will use the financial statement and the member's compilation report does not disclose a lack of independence; or c) an examination of prospective financial information. .02 This prohibition applies during the period in which the member is engaged to perform any of the services listed above and the period covered by any historical financial statements involved in such listed services. .03 Disclosure of permitted commissions. A member in public practice who is not prohibited by this rule from performing services for or receiving a commission and who is paid or expects to be paid a commission shall disclose that fact to any person or entity to whom the member recommends or refers a product or service to which the commission relates. .04 Referral fees. Any member who accepts a referral fee for recommending or referring any service of a CPA to any person or entity or who pays a referral fee to obtain a client shall disclose such acceptance or payment to the client. [Prior reference: paragraph .01 of ET section 503]

Zimmerman, a California CPA, is a manager with the firm Washington & Darrow, CPA's. Aimmerman is involved primarily with tax engagements but also helps out in the event that someone additional is needed on a financial statement audit that the firm is conducting. Zimmerman is neither a member of the AICPA or CAlCPA. Is Zimmerman bound by AICPA pronouncements? a. No. Only members of the AICPA must follow AICPA pronouncements. b. Yes. In many instances California licensees must adhere to AICPA pronouncements, whether or not they are members of the AICPA. c. Only if she issues financial statements. d. none of the above

Answer: Correct answer is B) Darrow is a CPA, but not a member of the AICPA or CalCPA. Is she bound by AICPA pronouncements? No. Only members of the AICPA must follow AICPA pronouncements. Yes. In many instances California licensees must adhere to AICPA pronouncements, whether or not they are members of the AICPA. Only if she issues financial statements. Only if she prepares tax returns and issues financial statements. a. is incorrect. Some pronouncements of the AICPA maybe applicable only to AICPA members. However, California licensees must comply with professional standards (including GAAP and GAAS) (Section 58), independence standards (Section 65) and reporting standards (Section 5062). b. is correct. The California Accountancy Act and the Accountancy Rules and Regulations require licensees to adhere to professional standards (Sections 5062, 58, 65).In many situations the applicable professional standards are promulgated by the AICPA. c. is incorrect. Whether or not financial statements are issued is not relevant to adherence to AICPA pronouncements. d. is incorrect. Whether or not financial statements or tax returns are issued is not relevant to adherence to AICPA pronouncements.

McGraw & Hill, CPAs are the auditors for a number of companies that are publicly owned and who file their financial statements with the Securities and Exchange Commission.Under the rules of the Public Company Accounting Oversight Board, which of the following services, performed during the audit and professional engagement period, would impair McGraw & Hill, CPAs independence with regard to any of these publicly owned companies who file their financial statements with the Securities and Exchange Commission? A) Provides any service or product to the audit client for a contingent fee or a commission, or receives from the audit client, directly or indirectly, a contingent fee or commission B) Provides any non-audit service to the audit client related to marketing, planning, or opining in favor of the tax treatment of, a transaction that is defined as an aggressive tax position transaction C) Provides any tax service to a person in a financial reporting oversight role at the audit client, or an immediate family member of such person D) All of the above would impair the firm's independence with these public owned companies.

Answer: D Rule 3521. Contingent Fees A registered public accounting firm is not independent of its audit client if the firm, or any affiliate of the firm, during the audit and professional engagement period, provides any service or product to the audit client for a contingent fee or a commission, or receives from the audit client, directly or indirectly, a contingent fee or commission. Rule 3522. Tax Transactions A registered public accounting firm is not independent of its audit client if the firm, or any affiliate of the firm, during the audit and professional engagement period, provides any non-audit service to the audit client related to marketing, planning, or opining in favor of the tax treatment of,a transaction— (a) Confidential Transactions—that is a confidential transaction; or (b) Aggressive Tax Position Transactions—that was initially recommended, directly or indirectly, by the registered public accounting firm and a significant purpose of which is tax avoidance, unless the proposed tax treatment is at least more likely than not to be allowable under applicable tax laws. Note 1: With respect to transactions subject to the United States tax laws, paragraph (b) of this rule includes, but is not limited to, any transaction that is a listed transaction within the meaning of 26 C.F.R. Sec. 1.6011(b)(2). Note 2: A registered public accounting firm indirectly recommends a transaction when an affiliate of the firm or another tax advisor, with which the firm has a formal agreement or other arrangement related to the promotion of such transactions, recommends engaging in the transaction. Rule 3523. Tax Services for Persons in Financial Reporting Oversight Roles A registered public accounting firm is not independent of an issuer audit client if the firm, or any affiliate of the firm, during the professional engagement period provides any tax service to a person in a financial reporting oversight role at the issuer audit client, or an immediate family member of such person, unless— (a) the person is in a financial reporting oversight role at the issuer audit client only because he or she serves as a member of the board of directors or similar management or governing body of the audit client; (b) the person is in a financial reporting oversight roleat the issuer audit client only because of the person's relationship to an affiliate of the entity being audited— (1) whose financial statements are not material to the consolidated financial statements of the entity being audited; or (2) whose financial statements are audited by an auditor other than the firm or an associated person of the firm; or (c) the person was not in a financial reporting oversight role at the issuer audit client before a hiring, promotion, or other change in employment event and the tax services are— (1) provided pursuant to an engagement in process before the hiring, promotion, or other change in employment event; and (2) completed on or before 180 days after the hiring or promotion event.

Denise, a member, is a sole practitioner with three professional staff and herself. Amador Holdings is a client of the firm and has been for the last year. Gary Amador, the owner of Amador Holdings, and Denise went to the same university and share a mutual passion for the football team.Gary has season tickets to the university football games and wants to offer his extra seats to Denise and her husband. While Denise want more than anything to attend, she is concerned that her objectivity and/or integrity might be impaired if she accepts the tickets.Which of the following factors would be relevant in determining if the entertainment is reasonable in the circumstances and not something that would normally impair objectivity and/or integrity? A) Whether the entertainment was associated with the active conduct of business either directly before, during, or after the entertainment B) The nature of the entertainment C) The nature of the entertainment D) All of the above are relevant factors.

Answer: D) all of the above .04 A member should evaluate the significance of any threats to determine if they are at an acceptablelevel. Threats are at an acceptable level when giftsor entertainment are reasonable in the circumstances. The member should exercise judgment in determining whether gifts or entertainment would be considered reasonable in the circumstances. The following are examples of relevant facts and circumstances: The nature of the gift or entertainment The occasion giving rise to the gift or entertainment The cost or value of the gift or entertainment The nature, frequency, and value of other gifts and entertainment offered or accepted Whether the entertainment was associated with the active conduct of business directly before, during, or after the entertainment Whether other customers or vendors also participated in the entertainment The individuals from the customer or vendor and a member's employer who participated in the entertainment

A person engaged in the practice of public accountancy is prohibited from performing services for a(an) ____for a commission or from receiving a commission from a client during the period in which the person also performs for that client an audit of a FS and during the period covered by any historical FS involved in the audit: client or an officer or director of a client or a client-sponsored retirement plan, all of the above

Answer: D) all of the above A person engaged in the practice of public accountancy is prohibited from performing services for a client, or an officer or director of a client, or a client-sponsored retirement plan, for a commission or from receiving a commission from a third party for providing the products or services of that third party to a client, or an officer or director of a client, or a client-sponsored retirement plan, during the period in which the person also performs for that client, or officer or director of that client, or client-sponsored retirement plan, any of the services listed below and during the period covered by any historical financial statements involved in those listed services: (1) An audit or review of a financial statement. (2) A compilation of a financial statement when that person expects, or reasonably might expect,that a third party will use the financial statement and the compilation report does not disclose a lack of independence. (3) An examination of prospective financial information. For purposes of this subdivision, "director" means any person as defined under Section 164 of the Corporations Code and "officer" means any individual reported to a regulatory agency as an officer of a corporation. However, "director" and "officer" does not include a director or officer of a nonprofit corporation, or a corporation that meets the board's definition of small business, as specified by regulation.

James Rome, a California CPA practicing in California, is a sole practitioner who has his office in San Diego. James provides various bookkeeping, accounting, payroll, and tax services to about 50 small business clients. Mary is an assistant to James and is the only employee of the firm. Which of the following is a proper form of notice to his clients regarding James' status as a Certified Public Accountant in the State of California? A) Displaying the certificate of licensure issued by the Board in the office or the public area of the premises where the licensee provides the licensed service. B) Any method of written notice, including a written notice that is electronically transmitted or a written notice posted at an Internet Website. C) Providing a statement to each client to be signed and dated by the client and retained in that person's records that states the client understands the person is licensed by the California Board of Accountancy. D) All of the above E) Both a. and c. but not b.

Answer: D) all of the above ARTICLE 9: Rules of Professional Conduct Section 50. Client Notification. Every licensee engaged in the practice of public accountancy shall provide notice reasonably calculated to be received by the licensee's clients of the fact that the licensee is licensed by the California Board of Accountancy. For purposes of this section, "licensee" means a Certified Public Accountant, Public Accountant, accountancy partnership, or accountancy corporation licensed by the California Board of Accountancy. Notice shall be provided by any of the following methods: (a) Displaying the certificate of licensure issued by the Board in the office or the public area of the premises where the licensee provides the licensed service.** (b) Providing a statement to each client to be signed and dated by the client and retained in that person's records that states the client understands the person is licensed by the California Board of Accountancy.** (c) Including a statement that the licensee is licensed by the California Board of Accountancy either on letterhead or on a contract for services where the notice is placed immediately above the signature line for the client in at least 12-point type. (d) Posting a notice in a public area of the premises where the licensee provides the licensed services,in at least 48-point type, that states the named licensee is licensed by the California Board of Accountancy. (e) Any other method of written notice, including a written notice that is electronically transmitted or a written notice posted at an Internet Website.**

Hayleah is a California CPA practicing in California. To renew her license in active status, Hayleah must to meet the basic requirement of completing 80 hours of qualifying continuing professional education in the two-year period immediately preceding her license expiration. Her license expires on June 30, 20X7. Which of the following are specific requirements regarding continuing education that Hayleah may or may not be covered by depending on the type of work that she performs? A) Fraud continuing education requirement B) Government auditing continuing education requirement C) Accounting and auditing continuing education requirement D) All of the above E) Hayleah may take all of her 80 hour requirement in any subjects that she chooses

Answer: D) all of the above Section 87. Basic Requirements. (a) 80 Hours. As a condition of renewing a license inan active status, a licensee shall complete at least 80 hours of continuing education in the two-year period immediately preceding license expiration, and meet the reporting requirements described in Section 89 (a). A licensee engaged in the practice of public accountancy as defined in Section 5051 of the Business and Professions Code is required to hold a license in an active status. No carryover of continuing education is permitted from one license renewal period to another. (1) A licensee renewing a license in an active status, shall complete a minimum of 20 hours in each year of the two-year license renewal period, with a minimum of 12 hours of the required 20 hours in technical subject areas as described in subsection (a)(2). **(2) Licensees shall complete a minimum of 50 percent of the required continuing education hours in the following technical subject areas: accounting, auditing, fraud, taxation, consulting, financial planning, ethics as defined in subsection (b), regulatory review as defined in Section 87.8, computer and information technology (except for word processing), and specialized industry or government practices that focus primarily upon the maintenance and/or enhancement of the public accounting skills and knowledge needed to competently practice public accounting. (3) Licensees may claim no more than 50 percentof the required number of continuing education hours in the following non-technical subject areas: communication skills, word processing, sales, marketing, motivational techniques, negotiation skills, office management, practice management, and personnel management.

Ross & Ross, CPAs performed an audit of the financial statements of Ruby Manufacturing for the year ended December 31, 20X7 and issued their report on March 2, 20X8. As of April 1, 20X8, due to various circumstances, Ross & Ross, CPAs was no longer independent with regard to Ruby Manufacturing and did not perform any further audits of the company's financial statements.On June 15, 20X8, Ross & Ross, CPAs was asked to re-sign the audited financial statements of Ruby Manufacturing for the year ended December 31, 20X7. Which of the following statements, if any, is true regarding this situation? A) Since Ross & Ross, CPAs are no longer independent with regard to Ruby Manufacturing, they may not re-sign the audit report. B) If post audit work was performed after March 2, 20X8, Ross & Ross, CPAs would not be able to re-sign the report. C) If post audit work was performed from April 1, 20X8 until June 15, 20X8, Ross & Ross, CPAs would not be able to re-sign the report. D) Since Ross & Ross, CPAs was independent at the time the report was issued, there is no restriction on their ability to re-sign the report even if their independence later becomes impaired.

Answer: D) can resign if not independent but not restate Change of Independent Auditors The auditor generally must be independent for the entire engagement period and the period covered by the financial statements being audited. Once this relationship is terminated, there is no continuing requirement for the auditor to remain independent. The auditor may generally re-issue its former opinions on the company's financial statements. However, if a restatement of the financial statements becomes necessary, the auditor must be independent to audit the restatement adjustments and re-issue its opinion. Further, if the Board is contemplating or plans a change in auditors, the audit committee must consider whether the prospective firm will be independent during the audit engagement period. That is, the prospective firm must cease all prohibited services and/or sever all prohibited relationships with the issuer prior to the beginning of the audit engagement period. Therefore, the audit committee should consider these issues before hiring a predecessor auditor or a prospective auditor to provide non-audit services to the company or its affiliates. Prospective firms can not audit financial statements of years that they were not independent.

Morton, a member, has a continuing attest engagement with Pirate Funding, Inc. Which of the following nonattest services would create a threat to independence that would be so significant that no safeguards could reduce the threat to an acceptable level? A) Acting as a consultant on a client's project in an advisory role B) Serving in an advisory role regarding the strategic direction for the client. C) Analyzing the client's system of internal control and providing recommendations for improvements. D) None of the above.

Answer: D) none of the above 1.295.105 Advisory Services .1 Self-review or management participation threats to compliance with the "Independence Rule" [1.200.001] may exist when a member performs advisory services for an attest client. .02 If the member's services are only advisory in nature and the member applies the "General Requirements for Performing Nonattest Services" interpretation [1.295.040] of the "Independence Rule" [1.200.001], threats would be at an acceptable level and independence would not be impaired. For example, a member may: a. provide advice, research materials, and recommendations to assist management in performing its functions and making decisions.** b. attend board meetings as a nonvoting advisor. c. interpret financial statements, forecasts, or other analyses. d. provide management with advice regarding its potential plans, strategies, or relationships.** .03 However,threatstocompliancewiththe"Independence Rule" [1.200.001] would not be at an acceptable level and could not be reduced to an acceptable level bythe application of safeguards if a member assumes any management responsibilities, as described inthe "Management Responsibilities" interpretation [1.295.030]. Accordingly, independence is impaired. [Prior reference: paragraph .05 of ET section 101 and paragraphs .015-.016 of ET section 191] 1.295.110 Appraisal, Valuation, and Actuarial Services .01 Self-review or management participation threats to compliance with the "Independence Rule" [1.200.001] may exist when a member performs appraisal, valuation, or actuarial service for an attest client.

Goldschmidt, a member, is a senior accountant with Diamondback Industries, a private company. Vargas is the controller of Diamondback Industries and is Goldschmidt's direct supervisor. While performing the year-end close for the period ended December 31, 20X5, Vargas has asked Goldschmidt to record several transactions with which Goldschmidt has some issues. All of the entries requested, if not correct, would materially overstate the net income of the company for the year in question.Which of the following steps should Goldschmidt undertake to ensure that the situation does not constitute a subordination of judgment? A) Goldschmidt should immediately resign his position and look for another job. B) Goldschmidt should immediately inform Diamondback Industries external accountant about the situation. C) Goldschmidt should immediately inform any and all third parties that might rely on the financial statements. D) None of the above E) Both a. and b.

Answer: D) none of the above first, go ti supervisor 2.130.020 Subordination of Judgment .01 The "Integrity and Objectivity Rule" [2.100.001] pro- hibits a member from knowingly misrepresenting facts or subordinating his or her judgment when performing professional services for an employer or on a volunteer basis. This interpretation addresses differences of opinion between a member and his or her supervisor or any other person within the member's organization. .04 In evaluating the significance of any identified threats, the member should determine, after appropriate research or consultation, whether the result of the position taken by the supervisor or other person a. fails to comply with professional standards, when applicable; b. creates a material misrepresentation of fact; or c. may violate applicable laws or regulations. .05 Ifthememberconcludesthatthreatsareatanacceptable level, the member should discuss his or her conclusions with the person taking the position. No further action would be needed under this interpretation. .06 If the member concludes that the position results ina material misrepresentation of fact or a violation of applicable laws or regulations, then threats would not be at an acceptable level. In such circumstances, the member should discuss his or her concerns with the supervisor. .07 Ifthedifferenceofopinionisnotresolvedafterdiscussing the concerns with the supervisor, the member should discuss his or her concerns with the appropriate higher level(s) of management within the member's organization (for example, the supervisor's immediate superior, senior management, and those charged with governance). .11 .12 .13 .14 .08 If after discussing the concerns with the supervisor and appropriate higher level(s) of management within the member's organization, the member concludes that appropriate action was not taken, then the member should consider, in no specific order, the following safeguards to ensure that threats to the member's compliance with the "Integrity and Objectivity Rule" [2.100.001] are eliminated or reduced to an acceptable level: Determine whether the organization's internal policies and procedures have any additional requirements for reporting differences of opinion. Determine whether he or she is responsible for communicating to third parties, such as regulatory authorities or the organization's (former organization's) external accountant. In considering such communications, the member should be cognizant of his or her obligations under the "Confidential Information Obtained From Employment or Volunteer Activities" interpretation [2.400.070] of the "Acts Discreditable Rule" [2.400.001] and the "Obligation of a Member to His or Her Employer's External Accountant" interpretation [2.130.030] of the "Integrity and Objectivity Rule" [2.100.001]. Consult with his or her legal counsel regarding his or her responsibilities. Document his or her understanding of the facts,the accounting principles, auditing standards, or other relevant professional standards involved or applicable laws or regulations and the conversations and parties with whom these matters were discussed. .09 If the member concludes that no safeguards can eliminate or reduce the threats to an acceptable levelor if the member concludes that appropriate action was not taken, then he or she should consider the continuing relationship with the member's organization and take appropriate steps to eliminate his or her exposure to subordination of judgment. .10 Nothing in this interpretation precludes a member from resigning from the organization at any time. However, resignation may not relieve the member of responsibilities in the situation, including any responsibility to disclose concerns to third parties, such as regulatory authorities or the employer's (former employer's) external accountant. .11 A member should use professional judgment and apply similar safeguards, as appropriate, to other situations involving a difference of opinion as described in this interpretation so that the member does not subordinate his or her judgment. [Prior reference: paragraph .05 of ET section 102] 2.130.030 Obligation of a Member to His or Her Employer's External Accountant .01 The "Integrity and Objectivity Rule" [2.100.001] requires a member to maintain objectivity and integrity in the performance of a professional service.When dealing with an employer's external accountant, a member must be candid and not knowingly misrepresent facts or knowingly fail to disclose material facts. This would include, for example, responding to specific inquiries for which the employer's external accountant requests written representation. [Prior reference: paragraph .04 of ET section 102]

Flim & Flam, CPAs is engaged to audit the financial statements of Rippin Off, Inc. for the year ending December 31, 20X4. The engagement letter for the audit was signed on August 31, 20X4 and some preliminary audit work began on that date.On November 1, 20X4, Val Capone, the CEO of Rippin Off, Inc. has asked Sam Flam, a partner at Flim & Flam, CPAs, if he would provide some corporate financial consulting services. Sam Flam does not want to perform any nonattest services related to corporate financial consulting if they would impair the firm's independence as it relates to the audit of Rippin Off, Inc. for the year ended December 31, 20X4Which of the following services could Sam provide that would not impair the firm's independence with regard to the audit engagement for Rippin Off, Inc.? A) Commit the client to the terms of a transaction B) Participate in transaction negotiations in an advisory capacity C) Assist in developing corporate financial strategies D) Acting as a promoter of client securities E) Both b. and c.

Answer: E) both b and c both are advisory capacity a is an overrstep of managerial function d is advertising Advisory services—practitioner develops findings, conclusions and recommendations for client decision making.

Taylor Underwood, a member, is a CPA who has her own accounting practice. Taylor is the sole owner of the practice and she has one non-CPA employee working as her accounting assistant. Which of the following names is Taylor permitted to use for her firm? A) Taylor Underwood & Company, Certified Public Accountants B) Taylor Underwood, Certified Public Accountants C) Taylor Underwood & Company, Certified Public Accountant D) None of the above are permitted

C) D)? Firm Names No firm title need name every owner. Such a requirement could become awkward and even unworkable. The firm may use the names of all or some of the owners: "Wilson and White" or "Finch, Fredericks and Smith." Or the firm may follow the name of one or more owners with designations "Company," "and Company," or "Associates." Thus, the firm "Finch, Fredericks and Smith" could choose instead to describe itself (among other possibilities) as "The David Finch Company," "David Finch and Company," "Finch, Fredericks and Associates," or "Finch, Fredericks and Company."

Gibbs, a member and CPA in public practice, wants to provide nonattest services to his attest client Harmon, Inc. Which of the following is not a general requirement for performing the nonattest services in order for Gibbs to maintain his independence with respect to Harmon, Inc.? A) Harmon, Inc. has agreed to evaluate the adequacy and results of the services performed. B) Harmon, Inc. has agreed to accept responsibility for the results of the services C) Harmon, Inc. has agreed to allow Gibbs to perform certain management functions as necessary in the performance of the nonattest services D) Both a. and b.

C) <checked>

The Conceptual Framework for AICPA Independence Standards contains definitions for various threats to independence. Which of the following threats, if any, are specifically mentioned and defined within the Conceptual Framework? A) Family threat B) Stock ownership threat C) Advocacy threat D) Both a. and b. E) Both b. and c.

C) Advocacy Advocacy threat. The threat that a member will promote an attest client's interests or position to the point that his or her independence is compromised. Examples of advocacy threats include the following: a. A member promotes the attest client's securities as part of an initial public offering. [1.295.130] b. A member provides expert witness services to an attest client. [1.295.140] c. A member represents an attest client in U.S. tax court or other public forum. [1.295.160]

Becker Co. is required to file its annual audited financial statements with the Government Accountability Office and Gearty & Green, CPA's have been hired to perform the engagement. When providing nonaudit services to Becker Co., which of the following understandings should be established and documented between Gearty & Green, CPAs and Becker Co.? A) Timing of the nonaudit services provided B) Cost of the nonaudit services provided. C) Any limitations of the nonaudit services provided D) Partner in charge of the nonaudit services provided.

C) Any limitations of the nonaudit services provided 3.39 In connection with nonaudit services, auditors should establish and document their understanding with the audited entity's management or those charged with governance, as appropriate, regarding the following: a. objectives of the nonaudit service; b. services to be performed; c. audited entity's acceptance of its responsibilities; d. the auditor's responsibilities; and e. any limitations of the nonaudit service.

Which of the following financial interests woudl impair a covered member's independence in such a manner that it could not be reduced to an acceptable level through the use of safeguards: - the covered member owned an immaterial direct financial interest in an attest client prior to but not during the period of the professional engagement - the covered member owned an immaterial indirect financial interest in an attest client during the period of the professional engagement - the covered member owned a material indirect financial interest in an attest client during the period of the professional engagement - the covered member owned a material indirect financial interest in an attest client prior to but not during the period of the professional engagement

C) the covered member owned a material indirect financial interest in an attest client during the period of the professional engagement prior to doesn't matter Immaterial indirect financial interests will not impair independence. Conversely, material indirect financial interests will impair independence. Since there is no materiality with respect to direct financial interests, the following investments of as little as one share of stock will impair independence:

Zagat, a member, is a partner in the firm Cruise & Cruise, CPA's. The firm handled the financial statement audit for Ships Ahoy, iNc. for the past three years but has now been informed that the client is moving to a new CPA firm. Zagat is reviewing the records request from Ships Ahoy to determine what records need to be released. As it related to records requests, an audit program is a: - client provided record - member prepared record - working paper - member's work product

C) working paper Section 1.400.200 (Records Requests)) says that client records are any accounting or other records belonging to the client. Conversely, a member's workpapers are those created by the member plus analyses and schedules prepared by the client at the member's request. Working papers are the licensee's records of the procedures applied, the tests performed, the information obtained and the pertinent conclusions reached in an audit, review, compilation, tax, special report or other engagement. They include, but are not limited to, audit of other programs, analyses, memoranda, letters of confirmation and representations, abstracts of company documents and schedules or commentaries prepared or obtained by the licensee. The form of working papers may be handwriting, typewriting, printing, photocopying, photographing, computer, data, or any other letters, words, pictures, sounds, or symbols or combinations thereof.

which of the following nonattest services, if any, are considered to be forensic accounting services? (a) Consultation regarding the preparation of wills and living trusts. (b) Tax preparation services for deceased taxpayers. (c) Investigative services (d) Preparing estate tax returns

C: Investigative services Explanation: Nonattest services are those that are not directly related to the completion of an attest engagement and are delivered to a client. Nonattest services have a broad reach, and the form of these services changes as clients' demands change. Tax return preparation, financial statement preparation, bookkeeping, certain forensic accounting services, such as investigative and some forms of litigation services, and appraisal, valuation, and actuarial services are some of the most generally cited examples. Nonattest services include: valuation, appraisal, and actuarial services; advisory services; payroll, bookkeeping, and other disbursements; benefit plan administration services; corporate finance consulting; business risk consulting; forensic accounting; executive or employee recruiting among others Reference: https://www.swflfraudatty.law/9-what-are-examples-of-nonattest-services

Bunnifer, a member, directly owns a 1% interest in an attest client. As it relates to independence, which of the following statements is true: a. Bunnifer's direct 1% interest in the attest client is immaterial and therefore the threat to her independence can be reduced to an acceptable level. b. Bunnifer's direct 1% interest in the attest client does not require safeguards to reduce the threat to her independence to an acceptable level because the direct interest is immaterial. c. Bunnifer's direct 1% interest in the attest client is such that the threat to her independence cannot be reduced to an acceptable level. d. None of the above.

Correct answer is c. a. is incorrect. A direct interest in a client, no matter how immaterial, is a threat to independence that cannot be reduced to an acceptable level. (Section 1.210.010.02) b. is incorrect. A direct interest in a client, no matter how immaterial, is a threat to independence that cannot be reduced to an acceptable level. (Section 1.210.010.02) d. is incorrect. As discussed above, answer option c. is a correct response. (Section 1.210.010.02)

Posey, a member, provides bookkeeping services for Giant Construction, a sole owner company with ten employees. On a monthly basis the owner sends Posey copies of check stubs and bank statements so that Posey can reconcile Giant's bank account and maintain the company's general ledger. At the end of the year, Posey prepares a trial balance based on the company's general ledger balances and utilizes that trial balance to prepare Giant Construction's required business tax returns and also prepares the owner's personal tax returns.Which of the following, as they relate to the services provided by Posey to Giant Construction, would impair Posey's independence? A) Preparation of the monthly bank reconciliation that identifies reconciling items that are not discussed or evaluated by the client. B) Coding checks that were not coded by the client and posting them to the company's general ledger without discussing or obtaining approval for the account coding from the client. C) Neither a. nor b. would impair independence because these are relatively common situations encountered when providing bookkeeping services to clients D) Both a. and b. would impair independence

D) Both a. and b. would impair independence 1.295.120 Bookkeeping, Payroll, and Other Disbursements .01 When a member provides bookkeeping, payroll, and other disbursement services to an attest client, self-review and management participation threats to the covered member's compliance with the "Independence Rule" [1.200.001] may exist. .02 If the member applies the "General Requirements for Performing Nonattest Services" interpretation [1.295.040] of the "Independence Rule" [1.200.001], threats would be at an acceptable level and independence would not be impaired. For example, a member may: a. record transactions to an attest client's general ledger when management has determined or approved the account classifications for the transaction. b. post client-coded transactions to an attest client's general ledger. c. prepare financial statements based on information in the attest client's trial balance. d. post client-approved journal or other entries to an attest client's trial balance. e. propose standard, adjusting, or correcting journal entries or other changes affecting the financial statements to the attest client. Prior to the member posting these journal entries or changes, the member should be satisfied that management has reviewed the entries and understands the nature of the proposed entries and the effect the entries will have on the attest client's financial statements. f. generate unsigned checks using source documents or other records provided and approved by the attest client. g. process an attest client's payroll using payroll time records that the attest client has provided and approved. h. transmit client-approved payroll or other disbursement information to a bank or similar entity subsequent to the attest client's review and authorization for the member to make the transmission. Prior to such transmission, the attest client is responsible for making the arrangements with the bank or similar entity to limit the corresponding individual payments regarding the amount and payee. In addition, once transmitted, the attest client must authorize the bank or similar entity to process the payroll information. i. prepare a reconciliation (for example, bank and accounts receivable) that identifies reconciling items for the client's evaluation. .03 However, threats to compliance with the"Independence Rule" [1.200.001] would not be at an acceptable level and could not be reduced to an acceptable level by the application of safeguards, and independence would be impaired, if, for example, a member: a. determines or changes journal entries, any account coding or classification of transactions, or any other accounting records without first obtaining the attest client's approval.** b. authorizes or approves transactions. c. prepares source documents. d. makes changes to source documents without the attest client's approval.** e. accepts responsibility to authorize payment of attest client funds, electronically or otherwise, except for electronic payroll tax payments when the member complies with the requirements of the "Tax Services" interpretation [1.295.160] of the "Independence Rule." f. accepts responsibility to sign or cosign an attest client's checks, even if only in emergency situations. maintains an attest client's bank account or otherwise has custody of an attest client's funds or makes credit or banking decisions for the attest client. approves vendor invoices for payment. [Prior reference: paragraph .05 of ET section 101]

Rollins, a member, is a partner in the firm Lee & Hamels, CPAs. The firm is heavily involved in providing attest services to local municipalities and local candidate's campaign organizations and has the requisite expertise to do so. Rollins is considered by many to be an expert in this particular field.Utley, a member of the Brotherly Love Party, is a client of the firm and wants to run for mayor of Philly City. Utley has asked Rollins to serve as treasurer of his campaign organization. Lee & Hamels, CPAs also provides attest services to Philly City and the Brotherly Love Party.If Rollins serves as the treasurer of Utley's campaign organization, with which of the following is the firm's independence impaired? A) Philly City B) Utley's campaign organization C) The Brotherly Love Party D) All of the above E) Both a and b

D) all of the above Political Party Is Attest Client .04 If during the period of the professional engagement or during the period covered by the financial statements a partner or professional employee serves as a campaign treasurer for a candidate and the political party for which the candidate is a member is an attest client, advocacy and familiarity threats may exist. Accordingly, a responsible individual within the firm should evaluate the significance of the threats to determine if the threats are at an acceptable level. If the responsible individual within the firm determines that threats are not at an acceptable level, he or she should apply safeguards to eliminate or reduce the threats to an acceptable level. However, threats would not be at an acceptable level and could not be reduced to an acceptable level by the application of safeguards and independence would be impaired if the candidate is a member of one of the political party's governing bodies.

Gracy, a member, is an audit manager with the firm of Gracy & Sogard, CPAs. The firm is engaged to prepare audited financial statements for Haber Machines for the year ended December 31, 20X4. Gracy's wife Sandra works as the executive assistant to the CEO of Haber Machines.As a result of her employment with Haber Machines, Sandra participates in the company's defined contribution pension plan. Through this participation in the Haber Machines defined contribution pension plan, Sandra has a direct financial interest in the company. Which of the following statements is true with regard to Gracy & Sogard, CPA's independence on the Haber Machines audit engagement? A) Independence is impaired in all cases. B) If Gracy does not participate in the attest engagement and is not in a position to influence the attest engagement, independence is not impaired. C) If Sandra's investment is an unavoidable consequence of her participation in the plan, independence is not impaired. D) None of the above E) Both b. and c. are required for independence to be maintained.

E) Both b. and c. are required for independence to be maintained. Ans: B?? .04 If a member fails to disassociate from the attest client before becoming a covered member, threats to the member's compliance with the "Independence Rule" [1.200.001] would not be at an acceptable level and independence would be impaired unless all of the following safeguards are met: The covered member ceases to participate in all employee health and welfare plans sponsored by the attest client, unless the attest client is legally required to allow the member to participate in the plan (for example, the Consolidated Omnibus Budget Reconciliation Act [COBRA]) and the member pays 100 percent of the member's portion of the cost of participation on a current basis. The covered member ceases to participate in all other employee benefit plans by liquidating or transferring, at the earliest date permitted underthe plan, all vested benefits in the attest client's defined benefit plans, defined contribution plans, share-based compensation arrangements, deferred compensation plans, and other similar arrangements. However, when a covered member's participation in one of these plans results from former employment or association with an attest client, threats would be at an acceptable level and independence would not be impaired provided the liquidation or transfer of any vested benefits is either not permitted under the terms of the plan or would result in a penalty significant to the benefits being imposed upon such liquidation or transfer and the covered member i. does not participate on the attest engagement team or ii.is not an individual in a position to influence the attest engagement.

Mariposa CPA's acquired Stockton CPA's in December 20X4. Mariposa is considered the acquiring firm and Stockton is considered the acquired firm. The combined firms will continue as Mariposa & Stockton CPA's after the acquisition. Which of the following is correct relating to independence in this situation: - if the acquiring firm provided prohibited nonattest services to an attest client of the acquired firm during the period covered by the FS, threats to compliance with the "Independence Rule" will not be at an acceptable level and cannot be reduced to an acceptable level by the application of safeguards. Accordingly, the acquiring firm's independence will be impaired with respect ot the attest client. - if the acquired firm provided prohibited nonattest services to an attest client of the acquiring firm prior to the FS period covered by the acquiring firm's next attest report, the acquiring firm's independence would be impaired - both a and b are correct - neither a nor b is correct

Prohibited Nonattest Services Provided by Acquiring Firm .05 If the acquiring firm provided prohibited nonattest services to an attest client of the acquired firm during the period covered by the financial statements, threats to compliance with the "Independence Rule" [1.200.001] will not be at an acceptable level and cannot be reduced to an acceptable level by the application of safeguards. Accordingly, the acquiring firm's independence WILL BE impaired with respect to the attest client. Prohibited Nonattest Services Provided by Acquired Firm .06 If the acquired firm provided prohibited nonattest services to an attest client of the acquiring firm prior to the financial statement period covered by the acquiring firm's next attest report, the acquiring firm's independence would NOT be impaired.****

Which of the following services are considered to be consulting services? Personal financial planning, Tax representation, Personal tax planning none of the above

a) Personal financial planning Step-by-step explanation From the multiple provided above, personal financial planning is a services that is considered to be a consulting service. This is because personal financial planning gives professional feedback to people or individuals for a fee hence it is referred to as a Consulting Service that provides leadership in finance, strategy and business development for the people and individuals investing in finance.

Monica, a member, performs various nonattest services for Chandler Industries, Which of the following would impair Monica's independence? A) Monica has accepted the responsibility to sign checks for Chandler Industries in emergency situations only but has yet to sign any checks. B) Monica prepares the tax returns for Chandler Industries. C) Chandler Industries presents Monica with account coded check stubs on a monthly basis and Monica posts these checks to the company's general ledger based on the client's accounting coding. D) Both a and b E) Both a and c

e. both a & c OG: A Before performing nonattest services, the member should establish and document in writing his or her understanding with the client (board of directors, audit committee,o r management as appropriate in the circumstances) regarding the following: a. Objectives of the engagement b. Services to be performed c. Client's acceptance of its responsibilities d. Member's responsibilities e. Any limitations of the engagement Tax services are another area of specialization for CPAs. Can an auditor provide tax services without sacrificing independence? Yes, providing tax services to a client does not normally impair independence with that client. .02 If the member applies the "General Requirements for Performing Nonattest Services" interpretation [1.295.040] of the "Independence Rule" [1.200.001], threats would be at an acceptable level and independence would not be impaired. For example, a member may a. record transactions to an attest client's general ledger when management has determined or approved the account classifications for the transaction. b. post client-coded transactions to an attest client's general ledger. c. prepare financial statements based on information in the attest client's trial balance. d. post client-approved journal or other entries to an attest client's trial balance. e. propose standard, adjusting, or correcting journal entries or other changes affecting the financial statements to the attest client. Prior to the member posting these journal entries or changes, the member should be satisfied that management has reviewed the entries and understands the nature of the proposed entries and the effect the entries will have on the attest client's financial statements. f. generate unsigned checks using source documents or other records provided and approved by the attest client. g. process an attest client's payroll using payroll time records that the attest client has provided and approved. h. transmit client-approved payroll or other disbursement information to a bank or similar entity subsequent to the attest client's review and authorization for the member to make the transmission. Prior to such transmission, the attest client is responsible for making the arrangements with the bank or similar entity to limit the corresponding individual payments regarding the amount and payee. In addition, once transmitted, the attest client must authorize the bank or similar entity to process the payroll information. i. prepare a reconciliation (for example, bank and accounts receivable) that identifies reconciling items for the client's evaluation. .03 However, threats to compliance with the "Independence Rule" [1.200.001] would not be at an acceptable level and could not be reduced to an acceptable level by the application of safeguards, and independence would be impaired, if, for example, a member a. determines or changes journal entries, any account coding or classification of transactions, or any other accounting records without first obtaining the attest client's approval. b. authorizes or approves transactions. c. prepares source documents. d. makes changes to source documents without the attest client's approval. e. accepts responsibility to authorize payment of attest client funds, electronically or otherwise, except for electronic payroll tax payments when the member complies with the requirements of the "Tax Services" interpretation [1.295.160] of the "Independence Rule." f. accepts responsibility to sign or cosign an attest client's checks, even if only in emergency situations. maintains an attest client's bank account or otherwise has custody of an attest client's funds or makes credit or banking decisions for the attest client. approves vendor invoices for payment. [Prior reference: paragraph .05 of ET section 101]

which of the following, if committed by a member, is NOT considered an act discreditable to the profession? - disclosure of questions and/or answers to the Uniform CPA Exam - filing a member's personal tax return on extension - determined by a court of competent jurisdiction to have violated US laws against sexual harassment - makes, or permits or directs another to make materially false and mislead entries int eh FS or records of an entity

filing a member's personal tax return on extension .01 A member shall be considered in violation of the "Acts Discreditable Rule" [2.400.001] if the member, by vir- tue of his or her negligence, does any of the following: Makes, or permits or directs another to make, materially false and misleading entries in the financial statements or records of an entity. Fails to correct an entity's financial statements that are materially false and misleading when the member has the authority to record an entry. Signs, or permits or directs another to sign,a document containing materially false and misleading information. [Prior reference: paragraph .05 of ET section 501]


संबंधित स्टडी सेट्स

GI Nursing: Intestinal Obstruction

View Set

ACLS: Pre-test (plus some extra q's)

View Set

NRSG 305 Practice Questions Exam 3

View Set

AEFS - U7B - Write sentences with CAN or CAN'T

View Set

Final (Ch.6-11) Management Ethics

View Set